0% found this document useful (0 votes)
140 views5 pages

Round 2

1. The document contains 5 problems with solutions involving inequalities relating positive variables a, b, c. 2. The first problem finds the maximum value of an expression involving a, b, c where a + b + c = √5. 3. The second problem proves an inequality relating terms involving a, b, c, and their sums. 4. The remaining problems use techniques like AM-GM, Holder's inequality, and rearrangement to prove other inequalities relating polynomials of a, b, c.

Uploaded by

Khanh Ha Thuc
Copyright
© © All Rights Reserved
We take content rights seriously. If you suspect this is your content, claim it here.
Available Formats
Download as PDF, TXT or read online on Scribd
0% found this document useful (0 votes)
140 views5 pages

Round 2

1. The document contains 5 problems with solutions involving inequalities relating positive variables a, b, c. 2. The first problem finds the maximum value of an expression involving a, b, c where a + b + c = √5. 3. The second problem proves an inequality relating terms involving a, b, c, and their sums. 4. The remaining problems use techniques like AM-GM, Holder's inequality, and rearrangement to prove other inequalities relating polynomials of a, b, c.

Uploaded by

Khanh Ha Thuc
Copyright
© © All Rights Reserved
We take content rights seriously. If you suspect this is your content, claim it here.
Available Formats
Download as PDF, TXT or read online on Scribd
You are on page 1/ 5

1 Problems in Round 2


1. Given a, b, c ≥ 0 satisfying a + b + c = 5, find the maximum value
of the following expression:

P = (a2 − b2 )(b2 − c2 )(c2 − a2 ).

Solution
WLOG assume c ≥ b ≥ a; so that we have

(a2 − b2 )(b2 − c2 )(c2 − a2 ) = (c2 − b2 )(c2 − a2 )(b2 − a2 ) ≤ b2 c2 (c2 − b2 ).



Also note that 5 = a + b + c ≥ b + c since a ≥ 0.
Now, using the AM-GM inequality we have
√ ! !2 √ ! !2
5 5
(c + b) · −1 ·c · + 1 b · (c − b)
2 2
(√ )5
5(b + c) √
≤ (c + b) ≤ 5;
5

So that we get √
P ≤ 5.
√ √ !
5 5
And hence we are done. Equality holds if and only if (a, b, c) = + 1; − 1; 0
2 2
and its cyclics. 

2. a, b, c > 0. Show that we have


r
ab + bc + ca + 1 3 3 (a + b)(b + c)(c + a)
+ ≥ 1.
(a + b + c + 1)2 8 abc
First Solution
Using AM-GM on the four terms in the left side, we have
r s
ab + bc + ca + 1 3 3 (a + b)(b + c)(c + a) (ab + bc + ca + 1)(a + b)(b + c)(c + a)
2
+ ≥44 .
(a + b + c + 1) 8 abc 83 abc(a + b + c + 1)2

Hence it is sufficient to check that

(ab + bc + ca + 1)(a + b)(b + c)(c + a) ≥ 2abc(a + b + c + 1)2 .

Multiplying both sides with 2; we are required to prove that


! !
X X
c(a + b) + 2 ab(a + b) + 2abc ≥ 4abc(a + b + c + 1)2 .
cyc cyc

1
In this form, it is a direct application of Cauchy-Schwarz inequality, since
! !
X X  √ √ 2
c(a + b) + 2 ab(a + b) + 2abc ≥ 2 abc(a + b + c) + 2 abc
cyc cyc

= 4abc(a + b + c + 1)2 .

Second Solution
We prove the inequality

(ab + bc + ca + 1)(a + b)(b + c)(c + a) ≥ 2abc(a + b + c + 1)2 .

Introducing the notations p = a + b + c; q = ab + bc + ca; r = abc we see that


our last inequality equivalents

(q + 1)(pq − r) ≥ 2r(p + 1)2 ;

Or
pq 2 + pq ≥ 3r + 2p2 r + 4pr + qr;
Which, after simple calculations, is equivalent to with
2  1 1 2
p q 2 − 3pr + (pq − 9r) + q (pq − 9r) + p q 2 + 3q − 18r ≥ 0.

3 3 9 9
But, using the obvious facts that pq ≥ 9r and q 2 ≥ 3pr; it is enough to prove
that
q 2 + 3q − 18r ≥ 0
Note that from AM- GM inequality we have
p √
q 2 + 3q ≥ 2 3q 3 ≥ 2 3 · 27r2 = 18r;

Because using AM-GM

q 3 = (ab + bc + ca)3 ≥ 27a2 b2 c2 = 27r2 .

Hence proved. Equality holds if and only if a = b = c = 1.

3. For a, b, c > 0 ∧ n ∈ {N − {1}} such that an + bn + cn = 3, prove


that
an+1 · b + bn+1 · c + cn+1 · a ≤ 3.
Solution
Letting x = an ; y = bn ; z = cn we see that the given inequality is equivalent to
√ √ √
x · n xy + y · n yz + z · n zx ≤ 3;

Lemma (V. Cirtoaje)


For a, b, c > 0; ∧a + b + c = 3 we have
 √ √ √ 
a ab + b bc + c ca ≤ 3;

2
proof
We rewrite the inequality as
 √ √ √ 
3 a ab + b bc + c ca ≤ (a + b + c)2 ;

Or equivalently:
X √ √ √ 2
a − 2 ab + bc − c + ca ≥ 0;
cyc

Which is perfectly true. Hence our lemma is proved. 


Note that using Hölder’s inequality we have
√ √ √ n  3 1 3 1 3 1
2
x · n xy + y · n yz + z · n zx ≤ (x + y + z)n−2 · x 2 y 2 + y 2 z 2 + z 2 x 2 .
 3 1 3 1 3 1
2
≤ 3n−2 · x 2 y 2 + y 2 z 2 + z 2 x 2
≤ 3n ;
Using the lemma for x, y, z.
And hence we are done. Equality holds if and only if a = b = c = 1.
Remark (Nguyen Huy Tung)
Let a, b, c > 0 satisfy an + bn + cn = 3, and n ≥ 2 is a natural number. Then
we also have the following inequality:
an+1 bn + bn+1 cn + cn+1 an ≤ 3.

4. Given positives a, b, c, show that


a3 + b3 + c3 ab + bc + ca ab bc ca
· 2
≥ 2
+ 2
+ .
abc (a + b + c) ab + 2c bc + 2a ca + 2b2
Solution
We can rewrite the original inequality into:
a3 + b3 + c3 ab + bc + ca X c2
· 2
≥3−2 .
abc (a + b + c) cyc
ab + 2c2

Or;
a3 + b3 + c3 ab + bc + ca X c2
· + 2 ≥ 3.
abc (a + b + c)2 cyc
ab + 2c2
But, from AM-GM and Cauchy-Schwarz inequality we have
a3 + b3 + c3 ab + bc + ca X c2
· + 2
abc (a + b + c)2 cyc
ab + 2c2
a3 + b3 + c3 ab + bc + ca (a + b + c)2
≥ · + 2 ·
abc (a + b + c)2 (ab + bc + ca) + 2(a2 + b2 + c2 )
s
(a3 + b3 + c3 )(ab + bc + ca)(a + b + c)4
≥33 2;
abc(a + b + c)2 (ab + bc + ca + 2a2 + 2b2 + 2c2 )

3
And therefore it is enough to check that
2
(a3 + b3 + c3 )(ab + bc + ca)(a + b + c)2 ≥ abc ab + bc + ca + 2a2 + 2b2 + 2c2 ;

Which, on using the known inequality which can be proved by AM-GM :

(a + b + c)(ab + bc + ca) ≥ 9abc;

Reduces to
2
9(a + b + c)(a3 + b3 + c3 ) ≥ ab + bc + ca + 2a2 + 2b2 + 2c2 .

But, using the Cauchy-Schwarz inequality we have


2
9(a + b + c)(a3 + b3 + c3 ) ≥ 9(a2 + b2 + c2 )2 = 3a2 + 3b2 + 3c2
2
≥ ab + bc + ca + 2a2 + 2b2 + 2c2 ;
And hence we are done. 

5. If a, b, c > 0 satisfy c(a − b) 6= 0; Prove that

a3 b3 c3 8(a + b)3 (b + c)3 (c + a)3


+ + + ≥ 4.
(b + c)3 (b + c)3 (c + a)3 (ac − bc)2 (a + b + c)5
Solution
We rewrite our original inequality as
a3 + 2(b + c)3 b3 + (c + a)3 c3 + (a + b)3 8(a + b)3 (b + c)3 (c + a)3
+ + + ≥ 8.
(b + c)3 (c + a)3 (a + b)3 (ac − bc)2 (a + b + c)5
On directly applying AM-GM we get
a3 + 2(b + c)3 b3 + (c + a)3 c3 + (a + b)3 8(a + b)3 (b + c)3 (c + a)3
+ + +
(b + c)3 (c + a)3 (a + b)3 (ac − bc)2 (a + b + c)5
s
8 {a3 + 2(b + c)3 } {b3 + (c + a)3 } {c3 + (a + b)3 }
≥44 ≥ 8;
(ac − bc)2 (a + b + c)5

⇐⇒ a3 + 2(b + c)3 b3 + (c + a)3 c3 + (a + b)3 ≥ 2(ac − bc)2 (a + b + c)5 .


  

But, applying Hölder’s inequality we have


(a + b + c)6
b3 + (c + a)3
 3
c + (a + b)3 ≥

.
16
And hence we are only required to prove that
 3
a + 2(b + c)3 (a + b + c) ≥ 32(ac − bc)2 .

Note that using a ≥ 0, again we have


 3
a + 2(b + c)3 (a + b + c) ≥ 2(b + c)4 ≥ 32b2 c2 ≥ 32(bc − ca)2 .

4
Hence we are done. Equality holds if and only if a = 0; b = c or b = 0; a = c.
Remark
We also have the following results given c(b − a) > 0; a, b, c ≥ 0:

a3 b3 c3 16(a + b)3 (b + c)3 (c + a)3


• 3
+ 3
+ 3
+ ≥4
(b + c) (c + a) (a + b) (bc − ca)(b2 + c2 )(a + b + c)5
a3 b3 c3 8(a + b)3 (b + c)5 (c + a)3
• 3
+ 3
+ 3
+ p ≥ 4.
(b + c) (c + a) (a + b) bc(b2 + c2 )(a + b + c)5 2(bc − ca)(b2 + c2 )

You might also like